Joe's primes

Algebra Level 5

Let x , y x, y be complex numbers satisfying

x + y = a , x y = b , \begin{aligned} x + y & = a, \\ xy &= b,\\ \end{aligned}

where a a and b b are positive integers from 1 to 100 inclusive. What is the sum of all possible distinct values of a a such that x 3 + y 3 x^3 + y^3 is a positive prime number?

This problem is posed by Joe T.

Details and assumptions

It is stated that x x and y y are complex numbers. They need not be positive integers.


The answer is 55.

This section requires Javascript.
You are seeing this because something didn't load right. We suggest you, (a) try refreshing the page, (b) enabling javascript if it is disabled on your browser and, finally, (c) loading the non-javascript version of this page . We're sorry about the hassle.

16 solutions

Guiping Xie
Jul 14, 2013

We notice that x 3 + y 3 = ( x + y ) 3 3 ( x + y ) ( x y ) x^3+y^3 = (x+y)^3-3(x+y)(xy) .

Therefore we substitute into a a and b b to get a 3 3 a b = p a^3-3ab=p for some prime p p By the rational root theorem, since a a and b b are integers, a a must be 1 1 or the prime p p .

Checking a = 1 a=1 we get 1 3 b = p 1-3b=p which does not work because p p and b b must be positive by the given conditions on the problem.

Therefore a = p a = p and we get the equation p 3 3 p b p = 0 p^3-3pb-p=0 or p 2 3 b 1 = 0 p^2-3b-1=0 which means p 2 = 3 b + 1 p^2=3b+1 . Looking at the squares in mod 3 3 we get that p p must be congruent to 1 1 or 2 2 mod 3 3 .

The restriction on b b is bounded from 1 1 to 100 100 so we see the max value a a can be is 17 17 . Listing all the primes congruent to 1 1 or 2 2 mod 3 3 and less than or equal to 17 17 we get 2 , 5 , 7 , 11 , 13 , 17 2,5,7,11,13,17 for a sum of 55 55

I think you mixed up the a's and b's at the end. It should be "the restriction on b b " rather than "the restriction on a a " and then "max value of a a is 17." Other than that, yeah bunny.

Sotiri Komissopoulos - 7 years, 11 months ago

Log in to reply

Yeah, you are right. Thanks.

I can't seem to edit it because I cannot find the icon

Guiping Xie - 7 years, 10 months ago

Log in to reply

I fixed it for you.

Daniel Liu - 6 years, 10 months ago

there's another typo there and it should be:

"we get that p p (or a a , no difference) must be congruent to 1 1 or 2 ( m o d 3 ) 2\pmod 3 "

mathh mathh - 6 years, 10 months ago
Ankush Tiwari
Jul 16, 2013

We notice that x 3 x^3 + y 3 y^3 = ( x + y ) 3 (x + y)^3 - 3 x y ( x + y ) 3xy(x+y)

Putting the values of a a and b b we have x 3 x^3 + y 3 y^3 = a ( a 2 3 b ) a(a^2-3b)

Now for this to be prime one of the factors must be 1 and the other must be a prime number.

Case 1:

a a = 1 and 1 3 b 1-3b is prime

this is not possible for positive values of b b

Case 2: a 2 3 b a^2 - 3b = 1 and a is prime

i.e 3 b + 1 \sqrt{3b +1} is prime .

the possible values of a a are 2 , 5 , 7 , 11 , 13 , 17 2 , 5, 7, 11, 13, 17 which have a sum of 55. 55.

i did with a dif method

Anubhav Singh - 7 years, 10 months ago
Anubhav Singh
Jul 19, 2013

In the problem the value of (x^3) + (y^3) is to be a prime. So first of all the value of ( x^3) + (y^3) is to be obtained from the given values. (x^3) + (y^3) = (x+y) ( (x^2)- xy + (y^2) )..= (a)x ( (a^2)- 3b )............(A) So, (a) x ( (a^2) - 3b ) is to be a positive prime. Note that a prime can be factored into 2 ways: (1 x the prime) or (prime x 1). So (a) x ( (a^2) -3b ) can be factored into 2 ways.

Case I:..............a=1 and (a^2) - 3b = p (the prime)......so 3b = (a^2) - p = 1-p.........Since b>=1........this can never be unless p<0........so this case can't be considered...........

Case II:......a=p ..........(a^2) - 3b =1.........(B).........note that from the second equation............ 3b = (a^2)-1 = (a+1) x (a-1)........................since b is an integer.........3 must divide R.H.S.........note that gcd( (a+1, a-1) ) = +-1 or +-2........so 3 has to to divide either a+1 or a-1 but not both.

First let 3 divide a+1......so a has to be of the form 3k+2 for k=0,1,2,3...................... Plugging in k=0,1,2,3...........the diff values of a are obtained. Putting this values in (B) ..,,,the permissible values of a for which b<100 are : 2, 5, 11 , 17

Second, let 3 divide a-1............so a will be of the form a=3k+1 for k=0,1,2.... and repeating the same way as above the permissible values of a are : 7, 13

So the sum of different values of a is 2 + 5 + 7 + 11 + 13 +17 =55

x 3 + y 3 = ( x + y ) 3 3 x y ( x + y ) x 3 + y 3 = a 3 3 a b \displaystyle x^3+y^3=(x+y)^3-3xy(x+y)\Rightarrow x^3+y^3=a^3-3ab = a ( a 2 3 b ) =a(a^2-3b) Since x 3 + y 3 = p x^3+y^3=p for some prime p p , a a or a 2 3 b a^2-3b have to be 1 1 . Checking when a = 1 a=1 , we get 1 3 b = p 1-3b=p , which is not possible because of the conditions specified. So a 2 3 b = 1 a^2-3b=1 which is equivalent to a 2 1 ( m o d 3 ) a^2\equiv 1\pmod3 . In addition, x 3 + y 3 = a ( 1 ) = a x^3+y^3=a(1)=a . By Fermat's Little Theorem, a 2 1 ( m o d 3 ) a^2\equiv 1\pmod3 is always true when a 3 k a\neq3k for any positive integer k k . So this problem just reduces down to finding all primes with a a and b b less than 100. Clearly a a grows faster than b b so we need to apply the restriction on b b . With b = 100 b=100 , we get a 2 300 = 1 a < = 301 a < = 17 a^2-300=1\Rightarrow a<=\sqrt{301}\Rightarrow a<=17 . Adding all the primes equal to or less than 17 that are not divisible by 3 we get 2 + 5 + 7 + 11 + 13 + 17 = 55 2+5+7+11+13+17=55

Piyal De
Jul 15, 2013

In the problem the value of (x^3) + (y^3) is to be a prime. So first of all the value of ( x^3) + (y^3) is to be obtained from the given values.
(x^3) + (y^3) = (x+y) ( (x^2)- xy + (y^2) )..= (a)x ( (a^2)- 3b )............(A) So, (a) x ( (a^2) - 3b ) is to be a positive prime. Note that a prime can be factored into 2 ways: (1 x the prime) or (prime x 1). So (a) x ( (a^2) -3b ) can be factored into 2 ways.

Case I:..............a=1 and (a^2) - 3b = p (the prime)......so 3b = (a^2) - p = 1-p.........Since b>=1........this can never be unless p<0........so this case can't be considered...........

Case II:......a=p ..........(a^2) - 3b =1.........(B).........note that from the second equation............ 3b = (a^2)-1 = (a+1) x (a-1)........................since b is an integer.........3 must divide R.H.S.........note that gcd( (a+1, a-1) ) = +-1 or +-2........so 3 has to to divide either a+1 or a-1 but not both.

First let 3 divide a+1......so a has to be of the form 3k+2 for k=0,1,2,3...................... Plugging in k=0,1,2,3...........the diff values of a are obtained. Putting this values in (B) ..,,,the permissible values of a for which b<100 are : 2, 5, 11 , 17

Second, let 3 divide a-1............so a will be of the form a=3k+1 for k=0,1,2.... and repeating the same way as above the permissible values of a are : 7, 13

So the sum of different values of a is 2 + 5 + 7 + 11 + 13 +17 =55

Anirudh Chauhan
Jul 14, 2013

x^3+y^3= (x+y)(x^2+y^2-xy)=a (a^2-3b)=P For P to be a prime a must be a prime and simultaneously a^2-3b must be equal to one. Putting the values of a starting from 2 to 17 (only prime numbers with both 2 and 17 inclusive). We will stop at 17 because after that b becomes greater than 100. At last add the solutions of a you get which will be equal to 55

Why can't we have a 2 3 b a^2 - 3b be a prime and a = 1 a = 1 ?

Calvin Lin Staff - 7 years, 11 months ago
Wilson Kan
Jul 19, 2013

x 3 + y 3 = ( x + y ) ( x 2 x y + y 2 ) = a ( a 2 3 b ) x^3+y^3 = (x+y)(x^2-xy+y^2) = a (a^2 - 3b)

If this is a positive prime, then either a = 1 a = 1 or a 2 3 b = 1 a^2 - 3b = 1 . If a = 1 a = 1 , a 2 3 b < 0 a^2 - 3b < 0 . So we must have the condition that a 2 3 b = 1 a^2 - 3b = 1 . As long as a 2 < 300 a^2 < 300 , a a is not divisible by 3 and a a is prime, it will satisfy the criteria in the problem. That leaves a = 2 , 5 , 7 , 11 , 13 , 17 a = 2, 5, 7, 11, 13, 17 .

We have: x 3 + y 3 = ( x + y ) ( x 2 + y 2 x y ) = a ( a 2 3 b ) x^3+y^3=(x+y)(x^2+y^2-xy)=a(a^2-3b) . Since x 3 + y 3 x^3+y^3 is a positive prime number, we have two case:

Case 1: a=1 and a 2 3 b a^2-3b is a positive prime number. This is impossible since a 2 3 b = 1 3 b < 0 a^2-3b=1-3b<0 when b 1 b \geq 1 .

Case 2: a 2 3 b = 1 a^2-3b=1 and a is a positive prime number. Because b 100 b \leq 100 , so a 2 301 a^2 \leq 301 or a 17 a \leq 17 . Also, a 2 1 a^2 \equiv 1 (mod 3). Therefore, possible values of a are: 2,5,7,11,13,17.

The answer is: 2+5+7+11+13+17=55

Atonu Mukherjee
May 20, 2014

x 3 + y 3 = ( x + y ) 3 3 x y ( x + y ) = a 3 3 a b = a ( a 2 3 b ) x^3 + y^3 = (x + y)^3 -3xy(x+y) = a^3 - 3ab = a(a^2 - 3b)

Now, a ( a 2 3 b ) a(a^2 - 3b) is a prime number so, we have 2 2 case :

Case 1 1 : a = 1 a = 1 and ( a 2 3 b ) (a^2 -3b) is a prime number . But it's impossible for b b .

Case 2 2 : a a is a prime number and ( a 2 3 b ) = 1 (a^2 - 3b) = 1

a 2 = 1 + 3 b \Rightarrow a^2 = 1+3b

a = 1 + 3 b \Rightarrow a = \sqrt{1+3b}

So, 1 + 3 b \sqrt{1+3b} is a prime number.

Now, the possible values of a a are 2 , 5 , 7 , 11 , 13 , 17 2,5,7,11,13,17 which have a sum of 55 \boxed {55} .

Thyago Capitanio
May 20, 2014

(x+y)^3 = x^3 + y^3 + 3xy(x+y) So: a^3 = x^3 + y^3 +3ba x^3 + y^3 = a(a^2 - 3b) (x^3 + y^3) is a prime number, so [a(a^2 - 3b)] is too The only ways of [a(a^2- 3b)] being a prime number is:

a=1 and [a^2 - 3b] is a prime number (1st hyphotesis) or [a^2 -3b] = 1 and a is a prime number.

1st hyphotesis : a = 1 and [1 - 3b] is a prime number. Impossible, because b is at least 1, so [1 - 3b] is no greater than (-2)

2nd hypohtesis: a is a prime number and [a^2 - 3b]=1 So 'a' has to be a prime, and a^2 = 3k +1 ; k is an integer a^2 - 3b = 3k +1 -3b = 0 => 3(k - b) = 0 => k = b a^2 = 3b +1 so a^2 is bigger than 4(3 +1) and smaller than 301(3.100 +1) squaring all the primes: 2^2 = 4 = 3.1 +1; It fits 3^2 = 9 = 3.3; It doesn't fit 5^2 = 25 = 3.8 +1; It fits 7^2 = 49 = 3.16 + 1; It fits 11^2 = 121 = 3.40 + 1; It fits 13^2= 169 = 3.56 +1; It fits 17^2 = 289 = 3.96 +1; It fits 19^2 = 361; It doesn't fit, because in this case a^2 is greater than 301.

Then, all the values of a are: {2,5,7,11,13,17}

So, the sum of possible values of a is: 2 + 5 + 7 + 11 + 13 +17 = 55

Tobby Satyarama
May 20, 2014

x + y x + y is a positive integer, therefore x and y are conjugates. if x = k + j i x = k + ji ,

a = 2 k b = k 2 + j 2 a = 2k b = k^2 + j^2

and

x 3 + y 3 = 2 k 3 6 k j 3 = 2 k ( k 2 3 j 2 ) = a ( a 2 3 b ) x^3 + y^3 = 2k^3 - 6kj^3 = 2k(k^2 - 3j^2) = a(a^2 - 3b)

For this to be a prime number, either one of the two factors must equal 1, and the other a prime. But clearly, a a cannot be 1; therefore a a must be prime and a 2 3 b = 1 a^2 - 3b = 1 . Since all squares of primes (except 3) are 1 ( m o d 3 ) \equiv 1 \pmod{3} , this boils down to finding all primes whose squares are less than 301, i.e. 2, 5, 7, 11, 13, 17.

James Aaronson
May 20, 2014

First, we observe that by the fundamental theorem of algebra, we have a pair of x and y for any a and b.

Now, x 3 + y 3 x^3 + y^3 = a 3 3 a b a^3 - 3ab . It is easy to determine that this is true by substituting in expressions for a and b.

So a ( a 2 3 b ) a(a^2-3b) is prime. Hence, either a = 1 a = 1 , so the expression is negative (a cannot be -1 since that is not in the required range, and 1-3b is negative for any valid b), or a 2 3 b = 1 a^2 - 3b = 1 and a is prime.

Checking mod 3, we see that a is not 3, and noting that if a 2 > 301 a^2 > 301 , then b > 100 b > 100 will hold, so the only values of a that will work are the primes less than 18 (other than 3). These are 2, 5, 7, 11, 13, 17 with a sum of 55.

Jason Martin
Jul 20, 2013

We first note that x 3 + y 3 = ( x + y ) ( x 2 x y + y 2 ) = a ( ( x 2 + 2 x y + y 2 ) 3 x y ) = a ( a 2 3 b ) x^3+y^3=(x+y)(x^2-xy+y^2)=a((x^2+2xy+y^2)-3xy)=a(a^2-3b) . Since primes only have factors 1 1 and p p , we know either a = 1 a=1 and 1 3 b = p 1-3b=p (where p is a prime), or a = p a=p and p 2 3 b = 1 p^2-3b=1 . Since no values of b b would give us 1 3 b = p 1-3b=p for a positive prime p, our only option is for a = p a=p and p 2 3 b = 1 p^2-3b=1 . For a = 2 , 5 , 7 , 11 , 13 , 17 a=2, 5, 7, 11, 13, 17 we have b = 1 , 8 , 16 , 56 , 96 b=1, 8, 16, 56, 96 respectively. Larger prime values of a a will make b > 100 b>100 .

Therefore, our answer is 2 + 5 + 7 + 11 + 13 + 17 = 55 2+5+7+11+13+17=55 .

Vickie Wang
Jul 19, 2013

Factoring x 3 + y 3 x^3 + y^3 , we have ( x + y ) ( x 2 x y + y 2 ) = a ( a 2 3 b ) (x+y)(x^2 - xy + y^2) = a(a^2-3b) . For this to be a positive prime, either a a must be 1 or a 2 3 b a^2 - 3b must be 1, and both must be positive integers.

a = 1 a = 1 cannot be a solution because it makes a 2 3 b a^2 - 3b negative, so a 2 3 b a^2 - 3b must be 1, yielding ordered pairs ( 2 , 1 ) , ( 5 , 8 ) , ( 7 , 16 ) , ( 11 , 40 ) , ( 13 , 56 ) (2,1), (5,8), (7,16), (11,40), (13,56) , and ( 17 , 96 ) (17,96) for a a and b b . A quick check reveals that for any ordered pair ( a , b ) (a,b) , there exist complex number solutions x x and y y . Thus, our sum is 2 + 5 + 7 + 11 + 13 + 17 = 55 2 + 5 + 7 + 11 + 13 + 17 = 55 .

Hieu Pham
Jul 19, 2013

x^3 + y^3 = (x+y)^3 - 3xy(x+y) = a^3 - 3ab = a(a^2 - 3b) which is divisible by a If x^3 + y^3 is a prime number then a=1 or a is a prime number and a^2-3b=1 Because prime numbers and b are positive so a can't be 1, then a must be prime and a^2 - 3b =1 (0< b < 101). Checking all the possible solutions we get a = 2,5,7,11,13,17 and 19

Nishant Sharma
Jul 16, 2013

We notice that x 3 x^{3} + y 3 y^{3} = ( x + y ) (x + y) ( x 2 + y 2 x y ) (x^{2} + y^{2} - xy)

= ( x + y ) (x + y) ( ( x + y ) 2 3 x y ) ((x + y)^{2} - 3xy)

= a a ( a 2 3 b ) (a^{2} - 3b)

Since x 3 x^{3} + y 3 y^{3} is a positive prime ( \big( Let's say P ) \big) , so a a and ( a 2 3 b ) (a^{2} - 3b) are the factors of it. But we know the only factors a prime can have is 1 1 and the prime itself. This leads to two cases:

\bullet a a = 1 1 and ( a 2 3 b ) (a^{2} - 3b) = P

\implies P = 1 3 b 1 - 3b < 0 <0 ( \big( Since 3 b 3b 3 \geq3 ) \big) , which is unacceptable for a prime so rejected

\bullet ( a 2 3 b ) (a^{2} - 3b) = 1 1 and a a = P

Since b b 100 \leq100

\implies a a \leq 301 \sqrt{301}

Now 17 2 {17}^{2} < < 301 301 < < 18 2 {18}^{2}

\implies a a 17 \leq17 and because a a is a prime, so a a assumes all prime values between 2 2 and 17 17 ( \big( both inclusive ) \big) . We should also keep in mind that \forall prime values of a a found above, b b \in { 1 , 2 , 3 , . . . . . . . . . , 100 } \{1, 2, 3, .........,100\} ( \big( This restriction eliminates 3 3 as a possible value of a a ) \big) .

So the sum we seek is 2 + 5 + 7 + 11 + 13 + 17 2 + 5 + 7 + 11 + 13 + 17 = 55 \boxed{55}

0 pending reports

×

Problem Loading...

Note Loading...

Set Loading...